LSAT and Law School Admissions Forum

Get expert LSAT preparation and law school admissions advice from PowerScore Test Preparation.

 Administrator
PowerScore Staff
  • PowerScore Staff
  • Posts: 8917
  • Joined: Feb 02, 2011
|
#23015
Complete Question Explanation

Assumption. The correct answer choice is (A)

The stimulus asserts that supplier attempts to reform public education have been unsatisfactory, so consumers much demand reforms. The stimulus concludes that school vouchers that allow parents to direct funding are the solution, and argues that underachieving schools will be forced to improve.

The argument makes a number of assumptions. First, it presumes that because one general strategy has not worked, a very specific alternate strategy must be the solution. Second, the stimulus ignores that in allowing parents to direct funding away from underachieving schools, the plan should not be expected to provide those schools the means of improvement. Third, the argument ignores a multitude of factors that could influence what a parent wants in a child's education.

We are asked to find the assumption required by the argument.

Answer choice (A): This is the correct choice. The "broadest" assumption the stimulus makes is that academics, rather than other factors, would primarily drive the demand for schools. If parents think that convenient locations and non-academic opportunities are more important than reasonable academic quality, they would likely use the vouchers just to make sure that they are not too put out by needing to get Junior to school, or to make sure that Junior, who isn't too bright after all, has a chance at being scouted for a badminton scholarship. Since the voucher plan is to improve academics, it is absolutely essential that parents would mostly demand academics rather than some other consideration.

Answer choice (B): The argument does not need to assume that the job market will reinforce demand for academic excellence. It could be true that even in the absence of reinforcement from the job market, parents would want to send their children to academically outstanding institutions. This choice might be helpful, but it does not represent a necessary condition, so this is not the ideal response to an assumption question.

Answer choice (C): The argument concerns overall academic excellence, not whether schools would have to choose similar or identical means of creating a result of excellence. The argument does not need to assume that there is a single best way to educate students, and you should not confuse the idea of excellence with the idea of "a single best way."

Answer choice (D): The argument does not need to assume that children would be able to provide input. As long as the parents can determine which schools are excellent and respond accordingly, the plan is functioning as stated in the stimulus, and no other input from children is necessary.

Answer choice (E): The argument does not need to assume that schools choose any particular method of improving their academic excellence. One school could choose to focus on literature, another on mathematics. Both schools could improve overall excellence, and parents might choose one or the other based on their children's skills and interests. So, it is not necessary to the plan that schools improve all of their offerings.
 smjones
  • Posts: 2
  • Joined: Oct 17, 2017
|
#40653
Hello!

I have a question about this one. I narrowed down my contenders to A and D and chose D. Looking back, I now see why A is the correct choice, but I did want to clarify something. Answer choice D looked attractive to me because of the last sentence in the stimulus, "To attract students, academically underachieving schools will be forced to improve their academic offerings." I thought since they're goal is to attract students, then maybe the students do have some sort of say in the school they attend. Does this sentence really mean the academically underachieving schools would improve their academic offerings to attract the parents who would ultimately send their kids (the students) to the school?

I know how specific the LSAT can be with their wording and how every word can have a different affect on the stimulus, so that's why I read so much into that last sentence.

Any help would be appreciated! Thank you!
 Francis O'Rourke
PowerScore Staff
  • PowerScore Staff
  • Posts: 471
  • Joined: Mar 10, 2017
|
#40689
That is an interesting understanding of this stimulus. I would first point to the third sentence of this stimulus which states the following:
  1. parents are given the vouchers
  2. parents should be allowed to choose
From this we know that in the speaker's proposal parents are the decision makers. That much is certain.

Then it looks like you want to ask if children can influence parents. That is possible. Can aunts and grandfathers influence parents? That too is possible. Can the family doctor or rabbi influence parents? None of these are ruled out, and they all are within the realm of reason, so they are possible.

The question however is asking us what is necessary for the argument to assume. It is possible that the speaker thinks that a great many people influence the decision. It is also possible that the speaker thinks of parents are sovereign decision-makers. What is certain is that parents have the final say over where to send their children. Certainty is what you want to look for in Assumption questions, not possibility or likelihood.

When selecting an answer for assumption questions, remember that you are saying that that answer choice is absolutely necessary for the argument to make sense. You are saying that there is no way that the speaker cannot believe this answer choice if they are making the above argument. You gave a good explanation for how it is possible that the speaker assumes answer choice (D), but it does not look like you thought the speaker needed to assume this.

Let me know if this helps :-D
 smjones
  • Posts: 2
  • Joined: Oct 17, 2017
|
#40743
Yes, that helps. Thanks!
 sa3334
  • Posts: 9
  • Joined: Jun 14, 2018
|
#49001
Hi,
Ans choice C sounds similar (to a certain extent) to A, except that C is too strong..."there is a single best way..." We know that the parents would prefer academic quality over any other factors, which is what A says, but I'm thinking that C is wrong because a preference on academic quality doesn't mean that it's the BEST WAY to educate students.

Can you please clarify if my way of thinking is correct? Also, as a tip, should we avoid too strong of an answer for assumption type questions?

Thank you!
SA
 Adam Tyson
PowerScore Staff
  • PowerScore Staff
  • Posts: 5153
  • Joined: Apr 14, 2011
|
#49690
Try the Negation Technique on answer C, SA, and you get "There is not a single best way to educate students." In other words, there can be many different approaches that all work equally well. Does that ruin the argument? Not at all! Some schools might choose one approach to improve and attract students, others would try other ways. Sounds wonderful and innovative!

For an Assumption question, the correct answer is something that the author must believe is true. For that reason, if it turns out to be untrue it should destroy the argument. The negation of answer C doesn't wreck the argument, so it is not a required assumption of the argument.

Negate answer A, though, and we have a real problem. What if parents will prefer sports opportunities and convenient locations over academics? If that's the case, then market pressures will NOT raise the level of education, but will depress it in favor of those other factors! That's the impact we want to see when we negate the correct answer to an Assumption question.

You should be suspicious of strong answers to assumption questions, but if the argument is a strong one the assumption could be, too. Don't reject an answer simply because it is strong, but tread carefully and try that negation approach. You;re right, though, that in this case "best" is too strong and not an essential element of the argument.

Try that Assumption Negation Technique on every Assumption question, SA, and you will knock them out of the park, because it is a winner ever time. Good luck!
 esther913
  • Posts: 23
  • Joined: Apr 13, 2019
|
#64630
Hi,
I need help with finding the conclusion.
Due to the conclusion indicator "Therefore", I thought the main conclusion of this argument was that "reforms must be demanded by consumers."
However, when I narrowed the answer choices to (A) and (E), both choices seemed to be more related to the last sentence of the stimulus ("To attract students, academically underachieving schools will be forced to improve their academic offerings").
Since (A) seems to defend an idea from the last sentence, does that make the last sentence the conclusion of this argument? :-?
Clarification would be greatly appreciated!
Thank you.
 Brook Miscoski
PowerScore Staff
  • PowerScore Staff
  • Posts: 418
  • Joined: Sep 13, 2018
|
#64692
Esther,

Conclusion indicators help you to look for the conclusion, but you still must be careful to consider how the statements in the stimulus are related to each other. In this case, the "therefore" sentence is pretty non-descript, and the next sentence carries on by explaining the reform that should be demanded. So you need to focus on the explanation of that reform.

Looking at it this way, the conclusion is that schools will be forced to improve academics to attract the parent's vouchers (the last sentence). The basis of the conclusion is simply that the parents will be able to choose where to spend their vouchers. The gap is between choice and academics--the stimulus does not show you that parents will base their choices on academics. The right choice will close that gap--and (A) does the job.
 esther913
  • Posts: 23
  • Joined: Apr 13, 2019
|
#64707
Thank you for the thorough explanation!
I was able to find the conclusion and understand why (A) is the right answer. Thank you :)
User avatar
 zl2886@nyu.edu
  • Posts: 1
  • Joined: Jun 14, 2022
|
#95820
Thank you for the explanation! However, I find that A is not convincing enough. Since this question is asking for a necessary assumption, then I assume that the answer should be a necessary assumption instead of a sufficient one. Indeed, the answer A guarantees that the conclusion is right; that if the parents prefer academics over sports and convenience, then the school would necessarily increase their academic efficiency.

But I think that A is a sufficient condition instead of a sufficient one. To prove this, I find the negation of A (sufficient condition) does not threaten the argument. The negation of A could be "parents tend to prefer academic over the quality of the cafeteria instead of preferring academic over the sports and location." Thus, the correct answer should not be A.

I do concede that the other choices should be eliminated. But I just find it hard to convince myself that A is a necessary condition. I would be really appreciated it if someone could offer some help on this!

Get the most out of your LSAT Prep Plus subscription.

Analyze and track your performance with our Testing and Analytics Package.